Strenghtening or weakening a plan

This topic has expert replies
Legendary Member
Posts: 768
Joined: Mon Nov 30, 2009 3:46 am
Thanked: 21 times
Followed by:7 members

Strenghtening or weakening a plan

by GMATMadeEasy » Wed Oct 27, 2010 2:46 pm
What is the best strategy for the strenghten/weaken questions involving future/current plans ? Some specifics to this particular question type ?

User avatar
GMAT Instructor
Posts: 2193
Joined: Mon Feb 22, 2010 6:30 pm
Location: Vermont and Boston, MA
Thanked: 1186 times
Followed by:512 members
GMAT Score:770

by David@VeritasPrep » Wed Oct 27, 2010 4:54 pm
The plan question is a popular question type on the GMAT. This a type that breaks down into three different categories.

Strengthen Plan

Weaken Plan

and Inference Plan

With the Strengthen and Weaken Plan question you will not find a conclusion in the stimulus. Instead there is an automatic conclusion, "the plan will achieve the goal" for strengthen -- and "the plan will not achieve the goal" for weaken questions.


If you can identify the goal and the plan then you will be ready to efficiently evaluate the answer choices.

Did you have a particular example of one of these questions that you wanted to look at?

David
Veritas Prep | GMAT Instructor

Veritas Prep Reviews
Save $100 off any live Veritas Prep GMAT Course

Legendary Member
Posts: 768
Joined: Mon Nov 30, 2009 3:46 am
Thanked: 21 times
Followed by:7 members

by GMATMadeEasy » Sat Oct 30, 2010 8:01 am
David thank you.

I can write some questions from OG, but idea is to know whether one should treat plan strengthen/weaken question diferent from normal strenghten to be more efficient or we can reat them as normal question ?

User avatar
GMAT Instructor
Posts: 2193
Joined: Mon Feb 22, 2010 6:30 pm
Location: Vermont and Boston, MA
Thanked: 1186 times
Followed by:512 members
GMAT Score:770

by David@VeritasPrep » Sat Oct 30, 2010 12:09 pm
Definitely treat them differently - while keeping a similar approach.

You see, the plan question does not have a nice conclusion in the stimulus. The conclusion is "the plan will achieve the goal." for a strengthen question and "the plan will not achieve the goal" for a weaken question.


Let me give you an example of a weaken question that I wrote and posted a few months:


The financial committee is concerned with recent abrupt swings in the various stock markets. Some of the one day gains and losses have been the largest in history and with computerized trading, tremendous changes can occur even within one hour. In order to contain this volatility across the stock exchanges, the financial committee has proposed quicker curbs at the major stock exchanges. Under this plan the trading of a particular stock will be halted on the major stock exchanges once that stock loses just 20% percent of its value in a period of three hours or less.

Which of the following, if true, most clearly points to the conclusion that the financial committee's plan will not result in less overall volatility in the stock exchanges?

A. The current curbs on trading at the major stock exchanges are credited with preserving some level of stability as the speed of trading has greatly increased.

B. Currency trading and commodity futures markets have much more volatility than major stock exchanges.

C. When trading of a particular stock is halted at the major stock exchanges the value of that stock often plummets at the smaller exchanges that do not have automatic halts on trading.

D. Computerized trading programs that react too quickly to small changes in the values of stocks are the cause of the increased volatility

E. Investors should understand that the stock markets are volatile and that investments may gain or lose value very quickly.



Do you see that on this question there is not really a conclusion in the stimulus? The automatic conclusion is - "the plan will not achieve the goal"

So, figure out what the plan is and what the goal is and plug into the automatic conclusion, "the plan will not achieve the goal. "

After you have worked the problem, My explanation for this one is at this link:

https://www.beatthegmat.com/stock-market ... 64944.html
Veritas Prep | GMAT Instructor

Veritas Prep Reviews
Save $100 off any live Veritas Prep GMAT Course

Senior | Next Rank: 100 Posts
Posts: 72
Joined: Tue Aug 10, 2010 8:36 pm
Thanked: 2 times

by winner's attitude » Sun Oct 31, 2010 12:31 am
Hi David,

Very convincing explanation.. can you pls give same kind of explanation with stratagy for normal strenthen / weaken / assumption questions , because these three appear the most in CR.

Thanks very much in advance

User avatar
GMAT Instructor
Posts: 2193
Joined: Mon Feb 22, 2010 6:30 pm
Location: Vermont and Boston, MA
Thanked: 1186 times
Followed by:512 members
GMAT Score:770

by David@VeritasPrep » Sun Oct 31, 2010 8:11 am
Sure "winner's attitude" (great name by the way)

Let's start with the assumption question type since that one is pretty tough....


Here is a link to an assumption question that I wrote:

https://www.beatthegmat.com/nuclear-powe ... 29-15.html



And here is a link to a question from a Veritas book where I give a different approach to Assumption

https://www.beatthegmat.com/an-odd-cr-fr ... 68008.html


Hope that helps!
Veritas Prep | GMAT Instructor

Veritas Prep Reviews
Save $100 off any live Veritas Prep GMAT Course

Legendary Member
Posts: 1404
Joined: Tue May 20, 2008 6:55 pm
Thanked: 18 times
Followed by:2 members

by tanviet » Thu Apr 12, 2012 11:43 pm
GMATMadeEasy wrote:David thank you.

I can write some questions from OG, but idea is to know whether one should treat plan strengthen/weaken question diferent from normal strenghten to be more efficient or we can reat them as normal question ?
Thank you David for your postings of CR. They are great. I have a question, pls, help.

Can I consider Plan question as causal reasoning question? I can summarize plan question as "do X, Get Y". The reason I ask you this question is that I see I can apply the process of doing Causal argument to the plan question.
In the causal reasoning question, I do as following:
- paraphrasing argument: X cause Y
- prephrasing answer: assumption is : there is no other cause. a weakener will be: there is another cause.
- going to answer choices, looking for a match between prephrased thing and an answer choice.

that work order can be apply to plan question. I think so. David, members, pls, comment on my thinking.

User avatar
GMAT Instructor
Posts: 2193
Joined: Mon Feb 22, 2010 6:30 pm
Location: Vermont and Boston, MA
Thanked: 1186 times
Followed by:512 members
GMAT Score:770

by David@VeritasPrep » Fri Apr 13, 2012 5:53 am
I think that it is better to state that "x will ACHIEVE y" instead of "x causes y." The difference is subtle and I think that you are certainly on the right track with your thinking...Cause and effect is closely related to plan questions (which I would refer to as "plan" and "goal").

Yet there is a difference - with Cause and Effect you know that the effect exists. For example, doctors give better attention to wealthier people. So that effect is something that is observed we know that it happens, the question is "why?" or "what is the cause?" It might be that the doctor thinks of wealthier people as more important, or they pay better, or the doctor just sees them more. These are possible causes of an effect that we know has occurred.

It is the opposite with a plan question. We know that the Plan exists (this is what you are equating with the cause) what we Do Not Know is whether the GOAL (or in your words effect) will be achieved. Do you see how this is exactly the opposite? Instead of knowing what the result is and looking for the cause, we know the plan and the result (or goal) is what we are hoping to achieve.

So for a plan question you would know that the plan is to provide free athletic memberships to people so that they can exercise more and the goal is that the people will lose weight. We then need to see if the plan will meet that goal.

Does that help?
Veritas Prep | GMAT Instructor

Veritas Prep Reviews
Save $100 off any live Veritas Prep GMAT Course

Legendary Member
Posts: 1404
Joined: Tue May 20, 2008 6:55 pm
Thanked: 18 times
Followed by:2 members

by tanviet » Sun Apr 15, 2012 12:22 am
David@VeritasPrep wrote:I think that it is better to state that "x will ACHIEVE y" instead of "x causes y." The difference is subtle and I think that you are certainly on the right track with your thinking...Cause and effect is closely related to plan questions (which I would refer to as "plan" and "goal").

Yet there is a difference - with Cause and Effect you know that the effect exists. For example, doctors give better attention to wealthier people. So that effect is something that is observed we know that it happens, the question is "why?" or "what is the cause?" It might be that the doctor thinks of wealthier people as more important, or they pay better, or the doctor just sees them more. These are possible causes of an effect that we know has occurred.

It is the opposite with a plan question. We know that the Plan exists (this is what you are equating with the cause) what we Do Not Know is whether the GOAL (or in your words effect) will be achieved. Do you see how this is exactly the opposite? Instead of knowing what the result is and looking for the cause, we know the plan and the result (or goal) is what we are hoping to achieve.

So for a plan question you would know that the plan is to provide free athletic memberships to people so that they can exercise more and the goal is that the people will lose weight. We then need to see if the plan will meet that goal.

Does that help?
great insights. Thank you David. I have another question, pls, help

I heard that there are 2 ways of strengthening/weakening an argument. Way 1, increase belief in or cast doubt on an assumption. Way 2, directly support/weaken the conclusion.
I check the OG books and see that Most of the CR problems belong to way 1. I do not see any problem of way 2.

Can you give us an example of way 2.

There are some CR problems about archeology, strengthener of which seems to belong to way 2. But I do not know.

Can you give me your idea on thin point.